- Thu Oct 19, 2017 12:20 pm
#40723
Complete Question Explanation
(The complete setup for this game can be found here: lsat/viewtopic.php?t=1626)
The correct answer choice is (C)
The question stem establishes that O is appointed to the appellate court. When O is assigned there, then the appellate court roster is full, and every other variable must be assigned to the trial court:
Accordingly, answer choice (C) is correct. Note that three of the incorrect answers address the H/P uncertainty (as would be expected), and the other incorrect answer improperly places J.
(The complete setup for this game can be found here: lsat/viewtopic.php?t=1626)
The correct answer choice is (C)
The question stem establishes that O is appointed to the appellate court. When O is assigned there, then the appellate court roster is full, and every other variable must be assigned to the trial court:
Accordingly, answer choice (C) is correct. Note that three of the incorrect answers address the H/P uncertainty (as would be expected), and the other incorrect answer improperly places J.
You do not have the required permissions to view the files attached to this post.